Xem bài viết đơn
Old 11-07-2012, 08:31 PM   #7
hansongkyung
+Thành Viên+
 
Tham gia ngày: Jan 2012
Đến từ: Han Tae Woong - IMO 1998
Bài gởi: 493
Thanks: 109
Thanked 417 Times in 241 Posts
Gửi tin nhắn qua Yahoo chát tới hansongkyung
Một cách khác dựa trên ý tưởng trên
Vì $a_2a_3...a_n =1$ nên tồn tại các số $x_2,x_3,...,x_n$ sao cho:
$a_2=\frac{x_2}{x_3}, a_3=\frac{x_4}{x_3},...,a_n=\frac{x_n}{x_2}$
Sử dụng bđt AM-GM ta có
$(\frac{x_i+x_{i+1}}{x_{i+1}})^k \ge \frac{i^{i}}{(i-1)^{i-1}
}\frac{x_{i}}{x_{i+1}}$
Cho $i$ chạy từ $2 \to n$ ta có ĐPCM

Việt Nam cố lên
[RIGHT][I][B]Nguồn: MathScope.ORG[/B][/I][/RIGHT]
 
hansongkyung is offline   Trả Lời Với Trích Dẫn
The Following 3 Users Say Thank You to hansongkyung For This Useful Post:
DuyLTV (11-07-2012), lexuanthang (13-07-2012), nguoibimat (12-07-2012)
 
[page compression: 9.05 k/10.08 k (10.24%)]